Effective Bertini

The name of the pictureThe name of the pictureThe name of the pictureClash Royale CLAN TAG#URR8PPP











up vote
6
down vote

favorite












Let $X$ be a smooth complex projective manifold, and $L$ an ample line bundle. By Bertini's Theorem, for every integer $q$ big enough there exists an open dense subset $U_qsubset |qL|$ such that every divisor $D$ in $U_q$ is smooth.



Warm up question: is the complement of $U_q$ always a divisor?



We can define a bigger open subset $V_qsubset |qL|$ as
$$
V_q:= ; textrms. t. ; (X,frac1qD) ; textrm is klt
$$



My question is: what is the dimension of the complement of $V_q$ ? (or at least can we bound its asymptotic in $q$ ? e.g. is it upper-bounded by $aq^dim X$ with $a$ a constant which is strictly smaller than the volume of $L$? )










share|cite|improve this question

















  • 3




    For all $qgeq q_0$, the complement of $U_q$ is a divisor whose degree is known, cf. mathoverflow.net/questions/165672/… Replace $mathcalO(1)$ in that answer by an appropriately positive tensor power of $L$, and use the asymptotic formula to find $q_0$.
    – Jason Starr
    Nov 20 at 14:41











  • Thanks! My main issue is with $V_q$.
    – Giulio
    Nov 20 at 14:43










  • Just to make things more explicit, I am interested in the case $q>>0$. Moreover, let me explain a little computation to justify my last guess about the complement of $V_q$. Take $X=mathbbP^2$, and $L=mathcalO(3)$. Fix a line $H$, multiplying by $(q+1)H$ we can embed $| mathcalO(2q-1)|$ into the complement of $V_q$. My guess is that this is kind of the biggest thing that can be outside $V_q$ (but I have no proof for the moment).
    – Giulio
    Nov 20 at 20:37















up vote
6
down vote

favorite












Let $X$ be a smooth complex projective manifold, and $L$ an ample line bundle. By Bertini's Theorem, for every integer $q$ big enough there exists an open dense subset $U_qsubset |qL|$ such that every divisor $D$ in $U_q$ is smooth.



Warm up question: is the complement of $U_q$ always a divisor?



We can define a bigger open subset $V_qsubset |qL|$ as
$$
V_q:= ; textrms. t. ; (X,frac1qD) ; textrm is klt
$$



My question is: what is the dimension of the complement of $V_q$ ? (or at least can we bound its asymptotic in $q$ ? e.g. is it upper-bounded by $aq^dim X$ with $a$ a constant which is strictly smaller than the volume of $L$? )










share|cite|improve this question

















  • 3




    For all $qgeq q_0$, the complement of $U_q$ is a divisor whose degree is known, cf. mathoverflow.net/questions/165672/… Replace $mathcalO(1)$ in that answer by an appropriately positive tensor power of $L$, and use the asymptotic formula to find $q_0$.
    – Jason Starr
    Nov 20 at 14:41











  • Thanks! My main issue is with $V_q$.
    – Giulio
    Nov 20 at 14:43










  • Just to make things more explicit, I am interested in the case $q>>0$. Moreover, let me explain a little computation to justify my last guess about the complement of $V_q$. Take $X=mathbbP^2$, and $L=mathcalO(3)$. Fix a line $H$, multiplying by $(q+1)H$ we can embed $| mathcalO(2q-1)|$ into the complement of $V_q$. My guess is that this is kind of the biggest thing that can be outside $V_q$ (but I have no proof for the moment).
    – Giulio
    Nov 20 at 20:37













up vote
6
down vote

favorite









up vote
6
down vote

favorite











Let $X$ be a smooth complex projective manifold, and $L$ an ample line bundle. By Bertini's Theorem, for every integer $q$ big enough there exists an open dense subset $U_qsubset |qL|$ such that every divisor $D$ in $U_q$ is smooth.



Warm up question: is the complement of $U_q$ always a divisor?



We can define a bigger open subset $V_qsubset |qL|$ as
$$
V_q:= ; textrms. t. ; (X,frac1qD) ; textrm is klt
$$



My question is: what is the dimension of the complement of $V_q$ ? (or at least can we bound its asymptotic in $q$ ? e.g. is it upper-bounded by $aq^dim X$ with $a$ a constant which is strictly smaller than the volume of $L$? )










share|cite|improve this question













Let $X$ be a smooth complex projective manifold, and $L$ an ample line bundle. By Bertini's Theorem, for every integer $q$ big enough there exists an open dense subset $U_qsubset |qL|$ such that every divisor $D$ in $U_q$ is smooth.



Warm up question: is the complement of $U_q$ always a divisor?



We can define a bigger open subset $V_qsubset |qL|$ as
$$
V_q:= ; textrms. t. ; (X,frac1qD) ; textrm is klt
$$



My question is: what is the dimension of the complement of $V_q$ ? (or at least can we bound its asymptotic in $q$ ? e.g. is it upper-bounded by $aq^dim X$ with $a$ a constant which is strictly smaller than the volume of $L$? )







ag.algebraic-geometry






share|cite|improve this question













share|cite|improve this question











share|cite|improve this question




share|cite|improve this question










asked Nov 20 at 14:29









Giulio

1,037516




1,037516







  • 3




    For all $qgeq q_0$, the complement of $U_q$ is a divisor whose degree is known, cf. mathoverflow.net/questions/165672/… Replace $mathcalO(1)$ in that answer by an appropriately positive tensor power of $L$, and use the asymptotic formula to find $q_0$.
    – Jason Starr
    Nov 20 at 14:41











  • Thanks! My main issue is with $V_q$.
    – Giulio
    Nov 20 at 14:43










  • Just to make things more explicit, I am interested in the case $q>>0$. Moreover, let me explain a little computation to justify my last guess about the complement of $V_q$. Take $X=mathbbP^2$, and $L=mathcalO(3)$. Fix a line $H$, multiplying by $(q+1)H$ we can embed $| mathcalO(2q-1)|$ into the complement of $V_q$. My guess is that this is kind of the biggest thing that can be outside $V_q$ (but I have no proof for the moment).
    – Giulio
    Nov 20 at 20:37













  • 3




    For all $qgeq q_0$, the complement of $U_q$ is a divisor whose degree is known, cf. mathoverflow.net/questions/165672/… Replace $mathcalO(1)$ in that answer by an appropriately positive tensor power of $L$, and use the asymptotic formula to find $q_0$.
    – Jason Starr
    Nov 20 at 14:41











  • Thanks! My main issue is with $V_q$.
    – Giulio
    Nov 20 at 14:43










  • Just to make things more explicit, I am interested in the case $q>>0$. Moreover, let me explain a little computation to justify my last guess about the complement of $V_q$. Take $X=mathbbP^2$, and $L=mathcalO(3)$. Fix a line $H$, multiplying by $(q+1)H$ we can embed $| mathcalO(2q-1)|$ into the complement of $V_q$. My guess is that this is kind of the biggest thing that can be outside $V_q$ (but I have no proof for the moment).
    – Giulio
    Nov 20 at 20:37








3




3




For all $qgeq q_0$, the complement of $U_q$ is a divisor whose degree is known, cf. mathoverflow.net/questions/165672/… Replace $mathcalO(1)$ in that answer by an appropriately positive tensor power of $L$, and use the asymptotic formula to find $q_0$.
– Jason Starr
Nov 20 at 14:41





For all $qgeq q_0$, the complement of $U_q$ is a divisor whose degree is known, cf. mathoverflow.net/questions/165672/… Replace $mathcalO(1)$ in that answer by an appropriately positive tensor power of $L$, and use the asymptotic formula to find $q_0$.
– Jason Starr
Nov 20 at 14:41













Thanks! My main issue is with $V_q$.
– Giulio
Nov 20 at 14:43




Thanks! My main issue is with $V_q$.
– Giulio
Nov 20 at 14:43












Just to make things more explicit, I am interested in the case $q>>0$. Moreover, let me explain a little computation to justify my last guess about the complement of $V_q$. Take $X=mathbbP^2$, and $L=mathcalO(3)$. Fix a line $H$, multiplying by $(q+1)H$ we can embed $| mathcalO(2q-1)|$ into the complement of $V_q$. My guess is that this is kind of the biggest thing that can be outside $V_q$ (but I have no proof for the moment).
– Giulio
Nov 20 at 20:37





Just to make things more explicit, I am interested in the case $q>>0$. Moreover, let me explain a little computation to justify my last guess about the complement of $V_q$. Take $X=mathbbP^2$, and $L=mathcalO(3)$. Fix a line $H$, multiplying by $(q+1)H$ we can embed $| mathcalO(2q-1)|$ into the complement of $V_q$. My guess is that this is kind of the biggest thing that can be outside $V_q$ (but I have no proof for the moment).
– Giulio
Nov 20 at 20:37











1 Answer
1






active

oldest

votes

















up vote
6
down vote













Regarding the warm up question: No (although for $q$ sufficiently large the answer is yes, as Jason Starr comments.) Let $X = mathbbP^1 times mathbbP^2$ and let $L= mathcalO(1,1)$. Write homogeneous coordinates on the first factor as $(u:v)$ and on the second factor as $(x:y:z)$. A divisor $D$ in $H^0(L)$ is of the form
$$u (ax+by+cz) + v (dx+ey+fz)=0.$$
$D$ is singular if and only if the matrix
$$beginbmatrix
a & b& c \ d & e & f \ endbmatrix$$

has rank $1$. (If this matrix has rank $2$, $D$ is isomorphic to $mathbbP^2$ blown up at a point, when the matrix drops rank, $D$ turns into the union of a $mathbbP^2$ and a $mathbbP^1 times mathbbP^1$.) The condition that this matrix drops rank is codimension $2$.



The complement of $U_q$ is called the dual variety to $X$. It is usually (no precise meaning attached) true that the dual of a smooth variety is a divisor. For a while, it was an open problem to characterize smooth projective toric varieties whose duals were not divisors; this was solved by Dickenstein, Feichtner and Sturmfels.






share|cite|improve this answer




















  • Thanks!! In general, I am more interested in the asymptotic values of $q$ (actually, for $q=1$, $L$ might even have no sections at all!!). And of course, I think $V_q$ is the hard part. Thanks!!
    – Giulio
    Nov 20 at 20:33










Your Answer





StackExchange.ifUsing("editor", function ()
return StackExchange.using("mathjaxEditing", function ()
StackExchange.MarkdownEditor.creationCallbacks.add(function (editor, postfix)
StackExchange.mathjaxEditing.prepareWmdForMathJax(editor, postfix, [["$", "$"], ["\\(","\\)"]]);
);
);
, "mathjax-editing");

StackExchange.ready(function()
var channelOptions =
tags: "".split(" "),
id: "504"
;
initTagRenderer("".split(" "), "".split(" "), channelOptions);

StackExchange.using("externalEditor", function()
// Have to fire editor after snippets, if snippets enabled
if (StackExchange.settings.snippets.snippetsEnabled)
StackExchange.using("snippets", function()
createEditor();
);

else
createEditor();

);

function createEditor()
StackExchange.prepareEditor(
heartbeatType: 'answer',
convertImagesToLinks: true,
noModals: true,
showLowRepImageUploadWarning: true,
reputationToPostImages: 10,
bindNavPrevention: true,
postfix: "",
imageUploader:
brandingHtml: "Powered by u003ca class="icon-imgur-white" href="https://imgur.com/"u003eu003c/au003e",
contentPolicyHtml: "User contributions licensed under u003ca href="https://creativecommons.org/licenses/by-sa/3.0/"u003ecc by-sa 3.0 with attribution requiredu003c/au003e u003ca href="https://stackoverflow.com/legal/content-policy"u003e(content policy)u003c/au003e",
allowUrls: true
,
noCode: true, onDemand: true,
discardSelector: ".discard-answer"
,immediatelyShowMarkdownHelp:true
);



);













 

draft saved


draft discarded


















StackExchange.ready(
function ()
StackExchange.openid.initPostLogin('.new-post-login', 'https%3a%2f%2fmathoverflow.net%2fquestions%2f315783%2feffective-bertini%23new-answer', 'question_page');

);

Post as a guest















Required, but never shown

























1 Answer
1






active

oldest

votes








1 Answer
1






active

oldest

votes









active

oldest

votes






active

oldest

votes








up vote
6
down vote













Regarding the warm up question: No (although for $q$ sufficiently large the answer is yes, as Jason Starr comments.) Let $X = mathbbP^1 times mathbbP^2$ and let $L= mathcalO(1,1)$. Write homogeneous coordinates on the first factor as $(u:v)$ and on the second factor as $(x:y:z)$. A divisor $D$ in $H^0(L)$ is of the form
$$u (ax+by+cz) + v (dx+ey+fz)=0.$$
$D$ is singular if and only if the matrix
$$beginbmatrix
a & b& c \ d & e & f \ endbmatrix$$

has rank $1$. (If this matrix has rank $2$, $D$ is isomorphic to $mathbbP^2$ blown up at a point, when the matrix drops rank, $D$ turns into the union of a $mathbbP^2$ and a $mathbbP^1 times mathbbP^1$.) The condition that this matrix drops rank is codimension $2$.



The complement of $U_q$ is called the dual variety to $X$. It is usually (no precise meaning attached) true that the dual of a smooth variety is a divisor. For a while, it was an open problem to characterize smooth projective toric varieties whose duals were not divisors; this was solved by Dickenstein, Feichtner and Sturmfels.






share|cite|improve this answer




















  • Thanks!! In general, I am more interested in the asymptotic values of $q$ (actually, for $q=1$, $L$ might even have no sections at all!!). And of course, I think $V_q$ is the hard part. Thanks!!
    – Giulio
    Nov 20 at 20:33














up vote
6
down vote













Regarding the warm up question: No (although for $q$ sufficiently large the answer is yes, as Jason Starr comments.) Let $X = mathbbP^1 times mathbbP^2$ and let $L= mathcalO(1,1)$. Write homogeneous coordinates on the first factor as $(u:v)$ and on the second factor as $(x:y:z)$. A divisor $D$ in $H^0(L)$ is of the form
$$u (ax+by+cz) + v (dx+ey+fz)=0.$$
$D$ is singular if and only if the matrix
$$beginbmatrix
a & b& c \ d & e & f \ endbmatrix$$

has rank $1$. (If this matrix has rank $2$, $D$ is isomorphic to $mathbbP^2$ blown up at a point, when the matrix drops rank, $D$ turns into the union of a $mathbbP^2$ and a $mathbbP^1 times mathbbP^1$.) The condition that this matrix drops rank is codimension $2$.



The complement of $U_q$ is called the dual variety to $X$. It is usually (no precise meaning attached) true that the dual of a smooth variety is a divisor. For a while, it was an open problem to characterize smooth projective toric varieties whose duals were not divisors; this was solved by Dickenstein, Feichtner and Sturmfels.






share|cite|improve this answer




















  • Thanks!! In general, I am more interested in the asymptotic values of $q$ (actually, for $q=1$, $L$ might even have no sections at all!!). And of course, I think $V_q$ is the hard part. Thanks!!
    – Giulio
    Nov 20 at 20:33












up vote
6
down vote










up vote
6
down vote









Regarding the warm up question: No (although for $q$ sufficiently large the answer is yes, as Jason Starr comments.) Let $X = mathbbP^1 times mathbbP^2$ and let $L= mathcalO(1,1)$. Write homogeneous coordinates on the first factor as $(u:v)$ and on the second factor as $(x:y:z)$. A divisor $D$ in $H^0(L)$ is of the form
$$u (ax+by+cz) + v (dx+ey+fz)=0.$$
$D$ is singular if and only if the matrix
$$beginbmatrix
a & b& c \ d & e & f \ endbmatrix$$

has rank $1$. (If this matrix has rank $2$, $D$ is isomorphic to $mathbbP^2$ blown up at a point, when the matrix drops rank, $D$ turns into the union of a $mathbbP^2$ and a $mathbbP^1 times mathbbP^1$.) The condition that this matrix drops rank is codimension $2$.



The complement of $U_q$ is called the dual variety to $X$. It is usually (no precise meaning attached) true that the dual of a smooth variety is a divisor. For a while, it was an open problem to characterize smooth projective toric varieties whose duals were not divisors; this was solved by Dickenstein, Feichtner and Sturmfels.






share|cite|improve this answer












Regarding the warm up question: No (although for $q$ sufficiently large the answer is yes, as Jason Starr comments.) Let $X = mathbbP^1 times mathbbP^2$ and let $L= mathcalO(1,1)$. Write homogeneous coordinates on the first factor as $(u:v)$ and on the second factor as $(x:y:z)$. A divisor $D$ in $H^0(L)$ is of the form
$$u (ax+by+cz) + v (dx+ey+fz)=0.$$
$D$ is singular if and only if the matrix
$$beginbmatrix
a & b& c \ d & e & f \ endbmatrix$$

has rank $1$. (If this matrix has rank $2$, $D$ is isomorphic to $mathbbP^2$ blown up at a point, when the matrix drops rank, $D$ turns into the union of a $mathbbP^2$ and a $mathbbP^1 times mathbbP^1$.) The condition that this matrix drops rank is codimension $2$.



The complement of $U_q$ is called the dual variety to $X$. It is usually (no precise meaning attached) true that the dual of a smooth variety is a divisor. For a while, it was an open problem to characterize smooth projective toric varieties whose duals were not divisors; this was solved by Dickenstein, Feichtner and Sturmfels.







share|cite|improve this answer












share|cite|improve this answer



share|cite|improve this answer










answered Nov 20 at 14:47









David E Speyer

105k8270531




105k8270531











  • Thanks!! In general, I am more interested in the asymptotic values of $q$ (actually, for $q=1$, $L$ might even have no sections at all!!). And of course, I think $V_q$ is the hard part. Thanks!!
    – Giulio
    Nov 20 at 20:33
















  • Thanks!! In general, I am more interested in the asymptotic values of $q$ (actually, for $q=1$, $L$ might even have no sections at all!!). And of course, I think $V_q$ is the hard part. Thanks!!
    – Giulio
    Nov 20 at 20:33















Thanks!! In general, I am more interested in the asymptotic values of $q$ (actually, for $q=1$, $L$ might even have no sections at all!!). And of course, I think $V_q$ is the hard part. Thanks!!
– Giulio
Nov 20 at 20:33




Thanks!! In general, I am more interested in the asymptotic values of $q$ (actually, for $q=1$, $L$ might even have no sections at all!!). And of course, I think $V_q$ is the hard part. Thanks!!
– Giulio
Nov 20 at 20:33

















 

draft saved


draft discarded















































 


draft saved


draft discarded














StackExchange.ready(
function ()
StackExchange.openid.initPostLogin('.new-post-login', 'https%3a%2f%2fmathoverflow.net%2fquestions%2f315783%2feffective-bertini%23new-answer', 'question_page');

);

Post as a guest















Required, but never shown





















































Required, but never shown














Required, but never shown












Required, but never shown







Required, but never shown

































Required, but never shown














Required, but never shown












Required, but never shown







Required, but never shown






Popular posts from this blog

How to check contact read email or not when send email to Individual?

Bahrain

Postfix configuration issue with fips on centos 7; mailgun relay